Question

Question 4: 4. Show that the following polynomials form a basis for P3 1 - x, 1-x2 1 +x _X 5. Show that the following matrice

0 0
Add a comment Improve this question Transcribed image text
Answer #1

: P Vector space for all palynomials degseo atmosi 3 fe atb+2+ dx To show;1+x, 1-, l-x2 for P3 a,b,cid eIR 1-x2J-&form{ a bas

Add a comment
Know the answer?
Add Answer to:
Question 4: 4. Show that the following polynomials form a basis for P3 1 - x,...
Your Answer:

Post as a guest

Your Name:

What's your source?

Earn Coins

Coins can be redeemed for fabulous gifts.

Not the answer you're looking for? Ask your own homework help question. Our experts will answer your question WITHIN MINUTES for Free.
Similar Homework Help Questions
  • Given are the polynomials P1:=1+ 2y + 3y?, P2 :=1+ 4y +9y?, Pz:=1+ 8y + 27y....

    Given are the polynomials P1:=1+ 2y + 3y?, P2 :=1+ 4y +9y?, Pz:=1+ 8y + 27y. To show that P1, P2, P3 € R2[y] are linearly independent, proceed as follows. (a) Find the images Vı := [PL]B, V2 := [P2]B and V3 := [P3]b in R3 of P1, P2 and P3 under the coordinate map with respect to the standard basis B = {1, y, yʻ} of R2[y]. (b) Form the matrix A = (v1 V2 V3] and find its...

  • Let V = M2x2 be the vector space of 2 x 2 matrices with real number...

    Let V = M2x2 be the vector space of 2 x 2 matrices with real number entries, usual addition and scalar multiplication. Which of the following subsets form a subspace of V? The subset of upper triangular matrices. The subset of all matrices 0b The subset of invertible matrices. The subset of symmetric matrices. Question 6 The set S = {V1, V2,v;} where vi = (-1,1,1), v2 = (1,-1,1), V3 = (1,1,-1) is a basis for R3. The vector w...

  • 1. Į 101 Show that the polynomials B = {1,-1, 2.2-r, r*) is a basis of the vector space P3 of all...

    1. Į 101 Show that the polynomials B = {1,-1, 2.2-r, r*) is a basis of the vector space P3 of all polynomials up to degree 3 2. [10] Find the coordinate vector [(x - 1)]B where B is the basis given in Question 1. 1. Į 101 Show that the polynomials B = {1,-1, 2.2-r, r*) is a basis of the vector space P3 of all polynomials up to degree 3 2. [10] Find the coordinate vector [(x -...

  • 45 points) Consider the following vectors in R3 2 0 0 2 2 Vi = 1...

    45 points) Consider the following vectors in R3 2 0 0 2 2 Vi = 1 ;02 31; V3 = 11:04 = -1 ; Us = 4 2 2 3 (c) Find a basis of R3 among V1, V2, V3, V4, V5, and call it basis V. (d) Is vs Espan{V1, V2, 03, 04}? Explain. (e) Find the coordinates of us with respect to the basis V.

  • sorry for the bad picture, and thanks for your assitance in advance :) Chapter 4, Section...

    sorry for the bad picture, and thanks for your assitance in advance :) Chapter 4, Section 4.5, Question 17 Find a basis for the subspace of that is spanned by the vectors V1 = (1, 0, 0), v2 = (1, 0, 1), V3 = (4, 0, 1), V4 = (0, 0, -2) O vi and v3 form a basis for span {V1, V2, V3, V4}. O v3 and V4 form a basis for span {V1, V2, V3, V4}. O vi...

  • Let p, (t) 6+t, P2(t) =t-3t, p3 (t) = 1 +t-2t. Complete parts (a) and (b) below. Use coordinate vectors to show that th...

    Let p, (t) 6+t, P2(t) =t-3t, p3 (t) = 1 +t-2t. Complete parts (a) and (b) below. Use coordinate vectors to show that these polynomials form a basis for P2. What are the coordinate vectors corresponding to p, p2, and pa? P- Place these coordinate vectors into the columns fa matrix A. What can be said about the matrix A? O A. The matrix A forms a basis for R3 by the Invertible Matrix Theorem because all square matrices are...

  • 15 points) Consider the following vectors in R3 0 0 2 V1 = 1 ; V2...

    15 points) Consider the following vectors in R3 0 0 2 V1 = 1 ; V2 = 3 ; V3 = 1] ; V4 = -1;V5 = 4 1 2 3 = a) Are V1, V2, V3, V4, V5 linearly independent? Explain. b) Let H (V1, V2, V3, V4, V5) be a 3 x 5 matrix, find (i) a basis of N(H) (ii) a basis of R(H) (iii) a basis of C(H) (iv) the rank of H (v) the nullity...

  • Problem 4. Let B = {V1, 02, 03} CR, where [3] [1] 01 = 12, 02...

    Problem 4. Let B = {V1, 02, 03} CR, where [3] [1] 01 = 12, 02 = 12103 = 1 [1] [2] 4.1. Show that the matrix A = (v1 V2 V3) E M3(R) is invertible by finding its inverse. Conclude that B is a basis for R3. 4.2. Find the matrices associated to the coordinate linear transformation T:R3 R3, T(x) = (2]B- and its inverse T-1: R3 R3. Use your answers to find formulas for the vectors 211 for...

  • 3 3 -16 -2 -5 12 4 1-12 Find the reduced row echelon form of the...

    3 3 -16 -2 -5 12 4 1-12 Find the reduced row echelon form of the matrix B 0 0 0 0 0 0 -16 12 -5 1, and v3 = 1-12 Let Vi 4 17 5 Decide whether the following statements are true or false. 2 The vectors vi, V2, and v span R. The vectors vi , V2 , and V3 are linearly independent. 3 3-16 В 1-2-5 4 -1 -12 Find the reduced row echelon form of...

  • Problem 4 A set of vectors is given by S = {V1, V2, V3} in R3...

    Problem 4 A set of vectors is given by S = {V1, V2, V3} in R3 where eV1 = 1 5 -4 7 eV2 = 3 . eV3 = 11 -6 10 a) [3 pts) Show that S is a basis for R3. b) (4 pts] Using the above coordinate vectors, find the base transition matrix eTs from the basis S to the standard basis e. Then compute the base transition matrix sTe from the standard basis e to the...

ADVERTISEMENT
Free Homework Help App
Download From Google Play
Scan Your Homework
to Get Instant Free Answers
Need Online Homework Help?
Ask a Question
Get Answers For Free
Most questions answered within 3 hours.
ADVERTISEMENT
ADVERTISEMENT
ADVERTISEMENT